5 votos

Maximizar una función inusual (Putnam 1996)

"El pescado", dijo, "yo los amo y los respeto mucho. Pero los voy a matar a muertos antes de que este día termine."

-- Ernest Hemingway, El Viejo y el Mar


Tengo, con diferentes grados de concentración, estado tratando de resolver este problema para la mayoría de los de esta noche:

Si requerimos $\{a_1, a_2, ... a_n \} = \{1, 2, ... n\}$, determinar el valor máximo de $a_1 a_2 + a_2 a_3 + \dots + a_{n-1}a_n + a_n a_1$ como una función de la $n$.

(Que es, ¿cómo podemos - por permuting la primera $n$ enteros positivos para formar una secuencia $\{a_n\}$ - maximizar la expresión anterior para arbitrario $n$?)

Mi pensamiento era que la dificultad fundamental con que el problema era la falta de simetría. Por lo tanto, mi enfoque de abajo fue motivada por tratar de introducir algunos.

Observe que $\sum_{k = 1}^n a_k^2 = \sum_{k = 1}^n k^2$, independientemente de lo que la secuencia de $\{a_k\}$ es. Ahora, hacemos una cuidada reordenamiento por completar las plazas. (En la primera y segunda líneas, estamos a sólo el ajuste de la notación sigma.) Si $S$ es nuestra expresión original,

\begin{eqnarray} 2 \sum_{k = 1}^n a_k^2 - 2S = 2 \sum_{k = 1}^n a_k^2 - 2 (a_1 a_2 + a_2 a_3 + \dots + a_{n-1}a_n + a_n a_1) \\ = \left( \sum_{k = 1}^{n-1} a_k^2 + a_{k+1}^2 \right) + (a_n^2 + a_1^2) - 2 (a_1 a_2 + a_2 a_3 + \dots + a_{n-1}a_n + a_n a_1) \\ = \left( \sum_{k = 1}^{n-1} a_k^2 + a_{k+1}^2 \right) + (a_n^2 + a_1^2) + \left( \sum_{k = 1}^{n-1} - 2 a_k a_{k+1} \right) - 2a_n a_1 \end{eqnarray}

$$= \sum_{k = 1}^{n-1} (a_k - a_{k+1})^2 + (a_n - a_1)^2 \qquad (*)$$

Ahora, si $2\left(\sum_{k = 1}^n k^2 - S\right) \ge m$ con igualdad para algunos arreglos $\{a_k\}$,$\sum_{k = 1}^n k^2 - \frac{m}{2} \ge S$; por lo tanto, $$\frac{(n)(n+1)(2n+1)}{6} - \frac{m}{2} \qquad(**)$$

sería nuestro máximo valor. Si podemos minimizar $(*)$ a continuación, desde la $m$ será este mínimo, hemos terminado.

Lo que este se ha reducido a está tratando de mostrar que $(*)$ es lo suficientemente amable expresión a minimizar. Aunque es difícil para mí para mostrar en la notación, me he encontrado con una heurística que sugiere que, por un determinado $n$,

$$\min (*) = m = (n-2)(2)^2 + 2(1)^2 = 4n - 6$$

Esto hace, de hecho, llevar a la solución correcta (la comprobación de la respuesta numérica).

Alguien me puede ayudar a demostrar que esta afirmación es correcta?

2voto

Calvin Lin Puntos 33086

Sugerencia: Demostrar por inducción $n\geq 3$. Muestran que agregar $n$ en la secuencia (en un lugar especial) aumentarían la suma por lo menos 4.

Esta solución se desprende naturalmente teniendo en cuenta el caso de igualdad.

Nota: El caso de $n=1, 2$ tendrá que tratarse por separado, ya que dan valores que no encajan en su secuencia.

i-Ciencias.com

I-Ciencias es una comunidad de estudiantes y amantes de la ciencia en la que puedes resolver tus problemas y dudas.
Puedes consultar las preguntas de otros usuarios, hacer tus propias preguntas o resolver las de los demás.

Powered by:

X